Gradient and finding the direction of maximum rate of change

Click For Summary
The discussion revolves around understanding the concept of the gradient and its relation to the direction of maximum rate of change in a function. Participants clarify that the gradient itself indicates this direction, typically expressed as a unit vector. There is confusion regarding the second question, with suggestions that the exercise may require a specific representation of the direction, possibly as a unit vector. The importance of checking the exponent in the denominator of the first function is also emphasized. Ultimately, the direction of maximum rate of change is confirmed to be represented by the gradient normalized to a unit vector.
Taylor_1989
Messages
400
Reaction score
14

Homework Statement


Hi guys, it a very simple question, but it causing me a great deal of confusion. The questions are as follows:

upload_2017-3-28_10-58-20.png

So I worked out the ans for one which I have displayed below. But what I don't understand is what they want from the second question. Because the way I see it, the direction of maximum rate of change is the gradient itself. Am I missing something here because I am really lost.

Homework Equations



The Attempt at a Solution


1: ##\nabla f=\frac{-xi}{(x^2+y^2+z^2)^3} -\frac{yj}{(x^2+y^2+z^2)^3}-\frac{zk}{(x^2+y^2+z^2)^3}##

##\nabla f = le^{lx+my+nz}i+me^{lx+my+nz}j+ne^{lx+my+nz}k##

2: I make the direction as follows: ##(-xi-yj-zk)## & ##li+mj+nk##

Is this correct or have I miss understood the question?
 
Physics news on Phys.org
Check the exponent in the denominator in the first f in part 1).
For part 2: I agree with you : ##-\bf\hat r## and ##(l,m,n)##.

[edit] perhaps for 'direction' the exercise composer wants to see ##\bf \hat r## ?
 
Taylor_1989 said:
But what I don't understand is what they want from the second question. Because the way I see it, the direction of maximum rate of change is the gradient itself.

?

The direction of maximum rate of change is the direction of the gradient. That generally means a unit vector,
 
BvU said:
Check the exponent in the denominator in the first f in part 1).
For part 2: I agree with you : ##-\bf\hat r## and ##(l,m,n)##.

[edit] perhaps for 'direction' the exercise composer wants to see ##\bf \hat r## ?
Thank you I did not put the 3/2 in thanks.
 
PeroK said:
The direction of maximum rate of change is the direction of the gradient. That generally means a unit vector,

when u say unit vector you mean ##\frac{\nabla f}{|\nabla f|}##?
 
Taylor_1989 said:
when u say unit vector you mean ##\frac{\nabla f}{|\nabla f|}##?

In this case, yes. If you look at the difference between the gradient and the direction if the gradient for the second function - the exponential - you'll see the point.
 
Question: A clock's minute hand has length 4 and its hour hand has length 3. What is the distance between the tips at the moment when it is increasing most rapidly?(Putnam Exam Question) Answer: Making assumption that both the hands moves at constant angular velocities, the answer is ## \sqrt{7} .## But don't you think this assumption is somewhat doubtful and wrong?

Similar threads

  • · Replies 8 ·
Replies
8
Views
2K
  • · Replies 8 ·
Replies
8
Views
1K
  • · Replies 4 ·
Replies
4
Views
2K
  • · Replies 6 ·
Replies
6
Views
2K
  • · Replies 9 ·
Replies
9
Views
2K
  • · Replies 13 ·
Replies
13
Views
4K
  • · Replies 6 ·
Replies
6
Views
2K
Replies
1
Views
1K
  • · Replies 8 ·
Replies
8
Views
2K
Replies
6
Views
2K